1
$\begingroup$

I wondered, inspired in a result mentioned from [1] (page 45), what should be the asymptotic behaviour of the sequence on assumption of the First Hardy–Littlewood conjecture

$$\sum_{\substack{\text{primes }p_n\leq x\\\text{such that }p_n+2\text{ is prime}}}(p_{n+1}-p_n)^2$$

as $x\to\infty$. Thus the summation is over the terms of the sequence A001359 from the OEIS, lesser of twin primes $$p_1=3, p_2= 5,p_3= 11, p_4=17,p_5= 29,\ldots$$

and we assume that the First Hardy–Littlewood conjecture is true.

A reference for the first Hardy–Littlewood conjecture is this section of Wikipedia.

I don't know if this exercise is in the literature.

Question. Deduce under the assumption that the First Hardy–Littlewood conjecture is true, what should be the asymptotic behaviour of $$\sum_{\substack{\text{primes }p_n\leq x\\\text{such that }p_n+2\text{ is prime}}}(p_{n+1}-p_n)^2$$ as $x\to\infty$. If it is in the literature, feel free to refer the reference and I try to search and read the result from the literature. Many thanks.

References:

[1] Richard Crandall and Carl Pomerance, Prime Numbers: A Computational Perspective, Second Edition, Springer (2005).

$\endgroup$
5
  • 1
    $\begingroup$ Isn't the sum just equal to 4 times the number of twin primes $\le x$? $\endgroup$
    – tj_
    Oct 3, 2019 at 9:07
  • 2
    $\begingroup$ @tj_ I was confused by the notation too, but I think $p_n$ enumerates not all primes, but rather just twin primes (or rather the smaller of the two in each pair) $\endgroup$
    – Wojowu
    Oct 3, 2019 at 9:09
  • 1
    $\begingroup$ @tj_ If you read my previous comment again, then no - if I understand OP correctly, $p_{n+1}$ is the next prime after $p_n$ such that $p_{n+1}+2$ is prime. $\endgroup$
    – Wojowu
    Oct 3, 2019 at 9:15
  • $\begingroup$ @Wojowu: I see, thanks for clarification. $\endgroup$
    – tj_
    Oct 3, 2019 at 9:18
  • $\begingroup$ @tj_ many thanks to you and the other user in comments. The sequence $p_n$ is the sequence A001359 from the OEIS: lesser of twin primes. $\endgroup$
    – user142929
    Oct 3, 2019 at 9:36

1 Answer 1

1
$\begingroup$

You want to estimate $x \to +\infty$: $$\sum_{\substack{\text{primes }p_n\leq x\\\text{such that }p_{n+1}+2\text{ is prime}}}(p_{n+1}-p_n)^2$$ Let $n\in 2\mathbb{N}$, and consider the 3 tuple $\mathcal{H}_3 = (0,n,n+2)$.

The 3-tuple $(0,n,n+2)$ is admissible iff $n = 1 \pmod 3$ or $n = 0 \pmod 3$.

Let $\pi_{\mathcal{H}_3}(x) = \#\{(p,p+n,p+n+2) \in \mathbb{P}^3 \, | \, p+n+2 \leq x\}$

The Hardy-Littlewood conjecture states that : $$\pi_{\mathcal{H}_3}(x) \sim \left(\displaystyle\prod_{\text{p prime}}\frac{1-\frac{w(\mathcal{H}_3, p)}{p}}{(1-\frac1p)^{3}}\right) \, \dfrac{x}{\log(x)^3}$$

Where $w(\mathcal{H}_3, p)$ is the number of distinct residues $\pmod p$ in $\mathcal{H}_3$.

We have $w(\mathcal{H}_3, 2) = 1$ and $w(\mathcal{H}_3, 3)=2$ and $p \geq 5 \implies w(\mathcal{H}_3, p)=3$

Then: $$\pi_{\mathcal{H}_3}(x) \sim \dfrac{9}{2} \left(\displaystyle\prod_{\substack{5 \leq p \\ \text{p prime}}}\frac{1-\frac{3}{p}}{(1-\frac1p)^{3}}\right) \, \dfrac{x}{\log(x)^3}$$

See that The k-tuple conjecture holds also for consecutive primes.

Then if $\pi_{\mathcal{H}_3}(x,n)=\#\{(p_i,p_{i+1},p_{i+2}) \, | \, p_{i+2}-p_{i+1}=2, p_{i+1}-p_i=n,p_{i+2}\leq x\}$ we have:

$$\pi_{\mathcal{H}_3}(x, n) \sim \dfrac{9}{2} \left(\displaystyle\prod_{\substack{5 \leq p \\ \text{p prime}}}\frac{1-\frac{3}{p}}{(1-\frac1p)^{3}}\right) \, \dfrac{x}{\log(x)^3}$$


We have $n = 0 \bmod 2$ and $\big( n = 0 \bmod 3 \text{ or } n = 1 \bmod 3 \big)$, then: $$n = 0 \bmod 6 \text{ or } n = 4 \bmod 6 \quad , n \neq 0$$

Suppose that $n \leq N(x)$.

($N(x)$ is the maximum gap that $p_{i+2}-p_{i+1}=2, p_{i+1}-p_i=n$ hold in the interval $[1,x]$)

Then we have:

$$\displaystyle\sum_{\substack{\text{primes }p_k\leq x\\\text{such that }p_{k+1}+2\text{ is prime}}}(p_{k+1}-p_k)^2 = \displaystyle\sum_{\substack{n \leq N(x) \\ n = 6k, k \in \mathbb{N}^*}}n^2 \cdot \pi_{\mathcal{H}_3}(x, n) + \displaystyle\sum_{\substack{n \leq N(x) \\ n = 6k+4, k \in \mathbb{N}}}n^2 \cdot \pi_{\mathcal{H}_3}(x, n)$$

Using Granville's conjecture $N(x) \leq (\alpha+o(1))\log(x)^2$ with $\alpha \geq 2 e^{-\gamma}$.

We have the asymptotic formula for $\pi_{\mathcal{H}_3}(x, n)$ not depending of $n$, Then: $$\displaystyle\sum_{\substack{\text{primes }p_k\leq x\\\text{such that }p_{k+1}+2\text{ is prime}}}(p_{k+1}-p_k)^2 \sim \pi^{\prime}_{\mathcal{H}_3}(x) \left( \displaystyle\sum_{\substack{n \leq N(x) \\ n = 6k, k \in \mathbb{N}^*}}n^2 + \displaystyle\sum_{\substack{n \leq N(x) \\ n = 6k+4, k \in \mathbb{N}}}n^2 \right)$$

With $\pi^{\prime}_{\mathcal{H}_3}(x) = \pi_{\mathcal{H}_3}(x, 4)$ as example.

$\endgroup$
3
  • $\begingroup$ Many thanks for your attention, I'm going to study your answer. $\endgroup$
    – user142929
    Oct 3, 2019 at 11:02
  • 1
    $\begingroup$ @user142929, i edit my post to answer all questions. good luck .. $\endgroup$ Oct 3, 2019 at 13:04
  • 1
    $\begingroup$ No problem about the edits, and I prefer the detailed answers as yours. My knowledge about these techniques is not the best, therefore there may be a delay of a few days before I am accepting an answer, many thanks again. $\endgroup$
    – user142929
    Oct 3, 2019 at 17:52

Your Answer

By clicking “Post Your Answer”, you agree to our terms of service and acknowledge you have read our privacy policy.

Not the answer you're looking for? Browse other questions tagged or ask your own question.